Ch 55 & 59 Exam

अब Quizwiz के साथ अपने होमवर्क और परीक्षाओं को एस करें!

The nurse is giving discharge instructions to the client with uric acid renal calculi. Which statement by the client indicates the client understands the prescribed diet? a) "I will eliminate milk and other dairy products from my diet." b) "I should limit my intake of meat and fish." c) "I should avoid raw fruits and vegetables." d) "Chocolate, spinach, and strawberries are not allowed."

"I should limit my intake of meat and fish." A low-purine diet is prescribed for the client with uric acid renal calculi. Organ meats, shellfish, anchovies, asparagus, and mushrooms are foods high in purine.

30. A 29-year-old patient has just been told that he has testicular cancer and needs to have surgery. During a presurgical appointment, the patient admits to feeling devastated that he requires surgery, stating that it will leave him "emasculated" and "a shell of a man." The nurse should identify what nursing diagnosis when planning the patient's subsequent care? A) Disturbed Body Image Related to Effects of Surgery B) Spiritual Distress Related to Effects of Cancer Surgery C) Social Isolation Related to Effects of Surgery D) Risk for Loneliness Related to Change in Self-Concept

A Feedback: The patient's statements specifically address his perception of his body as it relates to his identity. Consequently, a nursing diagnosis of Disturbed Body Image is likely appropriate. This patient is at risk for social isolation and loneliness, but there's no indication in the scenario that these diagnoses are present. There is no indication of spiritual element to the patient's concerns.

29. A patient has returned to the floor from the PACU after undergoing a suprapubic prostatectomy. The nurse notes significant urine leakage around the suprapubic tube. What is the nurse's most appropriate action? A) Cleanse the skin surrounding the suprapubic tube. B) Inform the urologist of this finding. C) Remove the suprapubic tube and apply a wet-to-dry dressing. D) Administer antispasmodic drugs as ordered.

B Feedback: The physician should be informed if there is significant leakage around a suprapubic catheter. Cleansing the skin is appropriate but does not resolve the problem. Removing the suprapubic tube is contraindicated because it is unsafe. Administering drugs will not stop the leakage of urine around the tube.

A physician explains to the patient that he has an inflammation of the Cowper glands. Where are the Cowper glands located?

Below the prostate, within the posterior aspect of the urethra

18. A nurse is providing an educational event to a local men's group about prostate cancer. The nurse should cite an increased risk of prostate cancer in what ethnic group? A) Native Americans B) Caucasian Americans C) African Americans D) Asian Americans

C Feedback: African American men have a high risk of prostate cancer; furthermore, they are more than twice as likely to die from prostate cancer as men of other racial or ethnic groups.

2. An uncircumcised 78-year-old male has presented at the clinic complaining that he cannot retract his foreskin over his glans. On examination, it is noted that the foreskin is very constricted. The nurse should recognize the presence of what health problem? A) Bowen's disease B) Peyronie's disease C) Phimosis D) Priapism

C Feedback: Phimosis is the term used to describe a condition in which the foreskin is constricted so that it cannot be retracted over the glans. Bowen's disease is an in situ carcinoma of the penis. Peyronie's disease is an acquired, benign condition that involves the buildup of fibrous plaques in the sheath of the corpus cavernosum. Priapism is an uncontrolled, persistent erection of the penis from either neural or vascular causes, including medications, sickle cell thrombosis, leukemic cell infiltration, spinal cord tumors, and tumor invasion of the penis or its vessels.

The most common presenting objective symptoms of a urinary tract infection in older adults, especially in those with dementia, include? a) Hematuria b) Change in cognitive functioning c) Back pain d) Incontinence

Change in cognitive functioning The most common objective finding is a change in cognitive functioning, especially in those with dementia, because these patients usually exhibit even more profound cognitive changes with the onset of a UTI. Incontinence, hematuria, and back pain are not the most common presenting objective symptoms.

A patient presents to the emergency department with paraphimosis. The physician is able to compress the glans and manually reduce the edema. Once the inflammation and edema subside, what is usually indicated?

Circumcision Circumcision is usually indicated after the inflammation and edema subside

A nurse is providing care for a patient who has recently been admitted to the postsurgical unit from PACU following a transuretheral resection of the prostate. The nurse is aware of the nursing diagnosis of Risk for Imbalanced Fluid Volume. In order to assess for this risk, the nurse should prioritize what action?

Closely monitoring the input and output of the bladder irrigation system

A 52-year-old patient is scheduled to undergo ileal conduit surgery. When planning this patient's discharge education, what is the most plausible nursing diagnosis that the nurse should address?

Deficient knowledge related to care of the ileal conduit The patient will most likely require extensive teaching about the care and maintenance of a new urinary diversion

A patient has a flaccid bladder secondary to a spinal cord injury. The nurse recognizes this patients high risk for urinary retention and should implement what intervention in the patients plan of care?

Double voiding After each voiding, the patient is instructed to remain on the toilet, relax for 1 to 2 minutes, and then attempt to void again in an effort to further empty the bladder.

Which of the following would be included in a teaching plan for a patient diagnosed with a urinary tract infection? a) Drink liberal amount of fluids b) Void every 4 to 6 hours c) Use tub baths as opposed to showers d) Drink coffee or tea to increase diuresis

Drink liberal amount of fluids Patients diagnosed with a UTI should drink liberal amounts of fluids. They should void every 2 to 3 hours. Coffee and tea are urinary irritants. The patient should shower instead of bathe in a tub because bacteria in the bath water may enter the urethra.

A female patient has been experiencing recurrent urinary tract infections. What health education should the nurse provide to this patient?

Drink liberal amounts of fluids. To increase fluids and flow of wastes in urine

A patient has undergone the creation of an Indiana pouch for the treatment of bladder cancer. The nurse identified the nursing diagnosis of "disturbed body image." How can the nurse best address the effects of this urinary diversion on the patient's body image?

Encourage the patient to speak openly and frankly about the diversion. Allowing the patient to express concerns and anxious feelings can help with body image, especially in adjusting to the changes in toileting habits

A nurse is caring for a 33-year-old male who has come to the clinic for a physical examination. He states that he has not had a routine physical in 5 years. During the examination, the physician finds that digital rectal examination (DRE) reveals stoney hardening in the posterior lobe of the prostate gland that is not mobile. The nurse recognizes that the observation typically indicates what?

Evidence of a more advanced lesion

Which of the following accounts for the majority of ureteral injuries? a) Sports injuries b) Unintentional injuries c) Gunshot wounds d) Knife wounds

Gunshot wounds Gunshot wounds account for 95% of ureteral injuries, which may range from contusions to complete transection. Unintentional injury to the ureter may occur during gynecologic or urologic surgery. Knife wounds and sports injuries do not account for the majority of ureteral injuries.

A nurse is teaching a 53-year-old man about prostate cancer. What information should the nurse provide to best facilitate the early identification of prostate cancer?

Have a digital rectal examination and prostate-specific antigen (PSA) test done yearly

The nurse is obtaining a health history from a client describing urinary complications. Which assessment finding is most suggestive of a malignant tumor of the bladder? a) Incontinence b) Hematuria c) Dysuria d) Frequency

Hematuria The most common first symptom of a malignant tumor is hematuria. Most malignant tumors are vascular; thus, abnormal bleeding can be a first sign of abnormality. The client then has symptoms of incontinence (a later sign), dysuria and frequency.

Resection of a patients bladder tumor has been incomplete and the patient is preparing for the administration of the first ordered instillation of topical chemotherapy. When preparing the patient, the nurse should emphasize the need to do which of the following?

Hold the solution in the bladder for 2 hours before voiding

An adolescent is identified as having a collection of fluid in the tunica vaginalis of his testes. The nurse knows that this adolescent will receive what medical diagnosis?

Hydrocele A hydrocele refers to a collection of fluid in the tunica vaginalis of the testes

A patient is undergoing diagnostic testing for a suspected urinary obstruction. The nurse should know that incomplete emptying of the bladder due to bladder outlet obstruction can cause what?

Hydronephrosis

Which type of incontinency refers to the involuntary loss of urine due to medications? a) Overflow b) Urge c) Iatrogenic d) Reflex

Iatrogenic Iatrogenic incontinence is the involuntary loss of urine due to medications. Reflex incontinence is the involuntary loss of urine due to hyperreflexia in the absence of normal sensations usually associated with voiding. Urge incontinence is the involuntary loss of urine associated with a strong urge to void that cannot be suppressed. Overflow incontinence is the involuntary loss of urine associated with overdistention of the bladder.

The nurse is caring for a client diagnosed with bladder cancer and requiring a cystectomy. The nurse overhears the physician instructing the client on the presence of a stoma with temporary pouch. In gathering information for the client, which urinary diversion would the nurse select? a) Ureterosigmoidostomy b) Ileal conduit c) Kock Pouch d) Indiana Pouch

Ileal conduit When the physician is discussing a stoma, the nurse recognizes that the client will have an ileal conduit which is a cetaceous urinary diversion. Both the Kock Pouch and Indiana Pouch are continent urinary diversions. The ureterosigmoidostomy connects with the rectum for urinary drainage.

A patient has had her indwelling urinary catheter removed after having it in place for 10 days during recovery from an acute illness. Two hours after removal of the catheter, the patient informs the nurse that she is experiencing urinary urgency resulting in several small-volume voids. What is the nurses best response?

Inform the patient that this is not unexpected in the short term and scan the patients bladder following each void.

A patient who is postoperative day 12 and recovering at home following a laparoscopic prostatectomy has reported that he is experiencing occasional dribbling of urine. How should the nurse best respond to this patients concern?

Inform the patient that urinary control is likely to return gradually.

A patient is 24 hours postoperative following prostatectomy and the urologist has ordered continuous bladder irrigation. What color of output should the nurse expect to find in the drainage bag?

Light pink The urine drainage following prostatectomy usually begins as a reddish pink, then clears to a light pink 24 hours after surgery.

A patient confides to the nurse that he cannot engage in sexual activity. The patient is 27 years old and has no apparent history of chronic illness that would contribute to erectile dysfunction. What does the nurse know will be ordered for this patient to assess his sexual functioning?

Nocturnal penile tumescence tests Nocturnal penile tumescence tests may be conducted in a sleep laboratory to monitor changes in penile circumference during sleep using various methods to determine number, duration, rigidity, and circumference of penile erections

A nurse who provides care in a long-term care facility is aware of the high incidence and prevalence of urinary tract infections among older adults. What action has the greatest potential to prevent UTIs in this population?

Limit the use of indwelling urinary catheters

A nurse is planning the postoperative care of a patient who is scheduled for radical prostatectomy. What intraoperative position will place the patient at particular risk for the development of deep vein thrombosis postoperatively?

Lithotomy position Elastic compression stockings are applied before surgery and are particularly important for prevention of deep vein thrombosis if the patient is placed in a lithotomy position during surgery

A nurse has been asked to speak to a local women's group about preventing cystitis. Which of the following would the nurse include in the presentation? a) Importance of urinating every 4 to 6 hours while awake b) Suggestion to take tub baths instead of showers c) Need to urinate after engaging in sexual intercourse d) Need to wear underwear made from synthetic material

Need to urinate after engaging in sexual intercourse Measures to prevent cystitis include voiding after sexual intercourse, wearing cotton underwear, urinating every 2 to 3 hours while awake, and taking showers instead of tub baths.

The nurse is caring for a patient who underwent percutaneous lithotripsy earlier in the day. What instruction should the nurse give the patient?

Notify the physician about cloudy or foul-smelling urine The patient should report the presence of foul-smelling or cloudy urine since this is suggestive of a UTI.

The nurse has tested the pH of urine from a patients newly created ileal conduit and obtained a result of 6.8. What is the nurses best response to this assessment finding?

Obtain an order to increase the patients dose of ascorbic acid

A patient with a recent history of nephrolithiasis has presented to the ED. After determining that the patients cardiopulmonary status is stable, what aspect of care should the nurse prioritize?

Pain management

Which of the following is the most common symptom of bladder cancer? a) Painless gross hematuria b) Altered voiding c) Back pain d) Pelvic pain

Painless gross hematuria Painless gross hematuria is the most common symptom of bladder cancer. Pelvic and back pain may occur with metastasis. Any alteration in voiding or change in the urine may indicate cancer of the bladder.

Which finding is an early indicator of bladder cancer? a) Nocturia b) Occasional polyuria c) Painless hematuria d) Dysuria

Painless hematuria Initially, as cancer cells destroy normal bladder tissue, bleeding occurs and causes painless hematuria. (Pain is a late symptom of bladder cancer.) Occasional polyuria may occur with diabetes mellitus or increased alcohol or caffeine intake. Nocturia commonly accompanies benign prostatic hypertrophy. Dysuria may indicate a urinary tract infection.

A patient is admitted to a hospital with a diagnosis of spastic, neurogenic bladder. The nurse is aware that the pathophysiology of this condition is primarily due to which of the following occurrences? a) Inability of the bladder muscle to contract forcefully b) Presence of a lower motor neuron lesion c) Bladder distended until overflow incontinence occurs d) Patient's inability to exert motor control

Patient's inability to exert motor control Neurogenic bladder dysfunction results from a lesion of the nervous system that results in urinary incontinence. Spastic bladder is caused by any spinal cord lesion above the voiding reflex. There is a loss of conscious sensation and control. A spastic bladder empties on reflex.

A patient has experienced occasional urinary incontinence in the weeks since his prostatectomy. In order to promote continence, the nurse should encourage which of the following?

Pelvic floor exercises

The nurse is caring for a postoperative client who has a Kock pouch. Nursing assessment findings reveal abdominal pain, absence of bowel sounds, fever, tachycardia, and tachypnea. The nurse suspects which of the following? a) Stoma ischemia b) Stoma retraction c) Peritonitis d) Postoperative pneumonia

Peritonitis Clinical manifestations of peritonitis include abdominal pain and distention, absence of bowel sounds, nausea and vomiting, fever, changes in vital signs.

A male patient, who is 82 years of age, suffers from urinary incontinence. Which of the following factors should the nurse assess before beginning a bladder training program for the patient? a) Smoking habits b) Physical and environmental conditions c) Occupational history d) History of allergies

Physical and environmental conditions It is essential to assess the patient's physical and environmental conditions before beginning a bladder training program, because the patient may not be able to reach the bathroom in time. During the bladder training program, a change in environment may be an effective suggestion for the patient. It is not so essential to assess the patient's history of allergy, occupation, and smoking habits before beginning a bladder training program.

Which of the following is a characteristic of a normal stoma? a) Painful b) Dry in appearance c) Pink color d) No bleeding when cleansing stoma

Pink color Characteristics of a normal stoma include a pink and moist appearance. It is insensitive to pain because it has no nerve endings. The area is vascular and may bleed when cleaned.

A nurse practitioner is assessing a 55-year-old male patient who is complaining of perineal discomfort, burning, urgency, and frequency with urination. The patient states that he has pain with ejaculation. The nurse knows that the patient is exhibiting symptoms of what?

Prostatitis Perineal discomfort, burning, urgency, frequency with urination, and pain with ejaculation is indicative of prostatitis

A 22-year-old male is being discharged home after surgery for testicular cancer. The patient is scheduled to begin chemotherapy in 2 weeks. The patient tells the nurse that he doesnt think he can take weeks or months of chemotherapy, stating that he has researched the adverse effects online. What is the most appropriate nursing action for this patient at this time?

Provide empathy and encouragement in an effort to foster a positive outlook

Which of the following terms is used to refer to inflammation of the renal pelvis? a) Urethritis b) Cystitis c) Interstitial nephritis d) Pyelonephritis

Pyelonephritis Pyelonephritis is an upper urinary tract inflammation, which may be acute or chronic. Cystitis is inflammation of the urinary bladder. Urethritis is inflammation of the urethra. Interstitial nephritis is inflammation of the kidney.

A woman comes to her health care provider's office with signs and symptoms of kidney stones. Which of the following should be the primary medical management goal? a) Determine the stone type. b) Relieve any obstruction. c) Relieve the pain. d) Prevent nephron destruction.

Relieve the pain. The immediate objective is to relieve pain, which can be incapacitating depending on the location of the stone.

To decrease glandular cellular activity and prostate size, an 83-year-old patient has been prescribed finasteride (Proscar). When performing patient education with this patient, the nurse should be sure to tell the patient what?

Report the planned use of dietary supplements to the physician

The nurse who is leading a wellness workshop has been asked about actions to reduce the risk of bladder cancer. What health promotion action most directly addresses a major risk factor for bladder cancer?

Smoking cessation

A patient with kidney stones is scheduled for extracorporeal shock wave lithotripsy (ESWL). What should the nurse include in the patients post-procedure care?

Strain the patients urine following the procedure

A 42-year-old woman comes to the clinic complaining of occasional urinary incontinence when she sneezes. The clinic nurse should recognize what type of incontinence?

Stress incontinence. Stress incontinence is the involuntary loss of urine through an intact urethra as a result of sudden increase in intra-abdominal pressure

A 57-year-old male comes to the clinic complaining that when he has an erection his penis curves and becomes painful. The patients diagnosis is identified as severe Peyronies disease. The nurse should be aware of what likely treatment modality?

Surgery Surgical removal of mature plaques is used to treat severe Peyronies disease

The nurse is educating a female patient with a UTI on the pharmacologic regimen for treatment. What is important for the nurse to instruct the patient to do? a) Take the antibiotic for 3 days as prescribed. b) Understand that if the infection reoccurs, the dose will be higher next time. c) Be sure to take the medication with grapefruit juice. d) Take the antibiotic as well as an antifungal for the yeast infection she will probably have.

Take the antibiotic for 3 days as prescribed. The trend is toward a shortened course of antibiotic therapy for uncomplicated UTIs, because most cases are cured after 3 days of treatment. Regardless of the regimen prescribed, the patient is instructed to take all doses prescribed, even if relief of symptoms occurs promptly. Although brief pharmacologic treatment of UTIs for 3 days is usually adequate in women, infection recurs in about 20% of women treated for uncomplicated UTIs.

A 75-year-old male patient is being treated for phimosis. When planning this patients care, what health promotion activity is most directly related to the etiology of the patients health problem?

Teaching the patient about the importance of hygiene

A patient who has recently undergone ESWL for the treatment of renal calculi has phoned the urology unit where he was treated, telling the nurse that he has a temperature of 101.1F (38.4C). How should the nurse best respond to the patient?

Tell the patient to report to the ED for further assessment

A public health nurse is teaching a health class for the male students at the local high school. The nurse is teaching the boys to perform monthly testicular self-examinations. What point would be appropriate to emphasize?

Testicular cancer is a highly curable type of cancer

The nurse is teaching a patient how to perform self-catheterization. Which of the following directions should the nurse include? a) The catheterization should occur 4 to 6 hours and before bedtime. b) The nurse uses nonsterile technique in the hospital setting. c) The catheter is rinsed with sterile normal saline after soaking in a cleaning solution. d) Peroxide is recommended for cleaning the urinary catheter.

The catheterization should occur 4 to 6 hours and before bedtime. The average daytime clean intermittent catheterization schedule is every 4 to 6 hours and just before bedtime. The catheter is rinsed with tap water after soaking in a cleaning solution. Either antibacterial soap or Betadine solution is recommended for cleaning urinary catheters at home. The nurse uses sterile technique in the hospital setting.

When a client with an indwelling urinary catheter wants to walk to the hospital lobby to visit with family members, the nurse teaches him how to do this without compromising the catheter. Which client action indicates an accurate understanding of this information? a) The client keeps the drainage bag below the bladder at all times. b) The client loops the drainage tubing below its point of entry into the drainage bag. c) The client sets the drainage bag on the floor while sitting down. d) The client clamps the catheter drainage tubing while visiting with the family.

The client keeps the drainage bag below the bladder at all times. To maintain effective drainage, the client should keep the drainage bag below the bladder; doing so allows the urine to flow by gravity from the bladder to the drainage bag. The client shouldn't lay the drainage bag on the floor because the bag could become grossly contaminated. The client shouldn't clamp the catheter drainage tubing because this impedes the flow of urine. To promote drainage, the client may loop the drainage tubing above — not below — its point of entry into the drainage bag.

When describing the types of bladder tumors that may occur, which type would the nurse identify as most common? a) Squamous cell carcinoma b) Adenocarcinoma c) Transitional cell carcinoma d) Papillary carcinoma

Transitional cell carcinoma The most common type of bladder tumor is a transitional cell carcinoma which develops in the bladder's epithelial lining. The tumors are classified as papillary or nonpapillary. Papillary lesions are superficial and extend outward from the mucosal layer. Nonpapillary tumors are solid growths that grow inward, deep into the bladder wall. This type is more likely to metastasize, usually to the lymph nodes, liver, lungs, and bone. Other types include squamous cell carcinoma and adenocarcinoma.

Which of the following is the procedure of choice for men with recurrent or complicated UTIs? a) CT scan b) MRI c) IV urogram d) Transrectal ultrasonography

Transrectal ultrasonography A transrectal ultrasonography is the procedure of choice for men with recurrent or complicated UTIs.

A client comes to the clinic for a follow-up visit. During the interview, the client states, "Sometimes when I have to urinate I can't control it, and do not reach the bathroom in time." The nurse suspects that the client is experiencing which type of incontinence? a) Urge b) Functional c) Stress d) Overflow

Urge Urge incontinence occurs when the client experiences the sensation to void but cannot control voiding in time to reach a toilet. Stress incontinence occurs when the client has an involuntary loss of urine that results from a sudden increase in intra-abdominal pressure. Overflow incontinence occurs when the client experiences an involuntary loss of urine related to an overdistended bladder; the client voids small amounts frequently and dribbles. Functional incontinence occurs when the client has function of the lower urinary tract but cannot identify the need to void or ambulate to the toilet.

A client is scheduled to undergo surgical creation of an ileal conduit. The primary nurse educates the client about surgery and the postoperative period. The nurse informs the client that many members of the health care team (including a mental health practitioner) will see him. A mental health practitioner should be involved in the client's care to: a) assess whether the client is a good candidate for surgery. b) assess suicidal risk postoperatively. c) evaluate the client's need for mental health intervention. d) help the client cope with the anxiety associated with changes in body image.

help the client cope with the anxiety associated with changes in body image. Many clients who undergo surgery for creation of an ileal conduit experience anxiety associated with changes in body image. The mental health practitioner can help the client cope with these feelings of anxiety. Mental health practitioners don't evaluate whether the client is a surgical candidate. None of the evidence suggests that urinary diversion surgery, such as creation of an ileal conduit, places the client at risk for suicide. Although evaluating the need for mental health intervention is always important, this client displays no behavioral changes that suggest intervention is necessary at this time.

The nurse is leading a workshop on sexual health for men. The nurse should teach participants that organic causes of erectile dysfunction include what? Select all that apply

x) Diabetes x) Testosterone deficiency x) Parkinsonism

A major goal when caring for a catheterized patient is to prevent infection. Select all the nursing actions that apply. a) Wash the perineal area with soap and water at least twice daily. b) Disconnect the tubing to collect urine samples. c) Empty the collection bag at least every 8 hours to reduce bacterial growth. d) Irrigate the catheter every 24 hours. e) Suspend the drainage bag off the floor.

• Empty the collection bag at least every 8 hours to reduce bacterial growth. • Suspend the drainage bag off the floor. • Wash the perineal area with soap and water at least twice daily. Never disconnect the tubing to collect samples, irrigate, or ambulate the patient since this will allow bacteria to enter the closed system. Drainage systems should have an aspiration or puncture port from which a specimen can be obtained. The drainage system should not be disconnected. See Box 28-8 in the text.

The nurse caring for a patient after urinary diversion surgery monitors the patient closely for peritonitis by assessing for which of the following? Select all that apply. a) Hyperactive bowel sounds b) Muscle flaccidity c) Leukocytosis d) Abdominal distention

• Leukocytosis • Abdominal distention The nurse should monitor the patient for the following signs and symptoms of peritonitis: leukocytosis, abdominal distention, absence of bowel sounds, fever, muscle rigidity, guarding, and nausea and vomiting.

The nurse is caring for a client who is scheduled for the creation of an ileal conduit. Which statement by the client provides evidence that client teaching was effective? a) "My urine will be eliminated through a stoma." b) "I will not need to worry about being incontinent of urine." c) "A catheter will drain urine directly from my kidney." d) "My urine will be eliminated with my feces."

"My urine will be eliminated through a stoma." An ileal conduit is a non-continent urinary diversion whereby the ureters drain into an isolated section of ileum. A stoma is created at one end of the ileum, exiting through the abdominal wall.

A client with urinary tract infection is prescribed phenazopyridine (Pyridium). Which of the following instructions would the nurse give the client? a) "This will kill the organism causing the infection." b) "This medication should be taken at bedtime." c) "This medication will prevent re-infection." d) "This medication will relieve your pain."

"This medication will relieve your pain." Phenazopyridine (Pyridium) is a urinary analgesic agent used for the treatment of burning and pain associated with UTIs.

A client undergoes extracorporeal shock wave lithotripsy. Before discharge, the nurse should provide which instruction? a) "Increase your fluid intake to 2 to 3 L per day." b) "Apply an antibacterial dressing to the incision daily." c) "Take your temperature every 4 hours." d) "Be aware that your urine will be cherry-red for 5 to 7 days."

"Increase your fluid intake to 2 to 3 L per day." The nurse should instruct the client to increase his fluid intake. Increasing fluid intake flushes the renal calculi fragments through — and prevents obstruction of — the urinary system. Measuring temperature every 4 hours isn't needed. Lithotripsy doesn't require an incision. Hematuria may occur for a few hours after lithotripsy but should then disappear.

34. A patient has experienced occasional urinary incontinence in the weeks since his prostatectomy. In order to promote continence, the nurse should encourage which of the following? A) Pelvic floor exercises B) Intermittent urinary catheterization C) Reduced physical activity D) Active range of motion exercises

A Feedback: Pelvic floor muscles can promote the resumption of normal urinary function following prostate surgery. Catheterization is normally unnecessary, and it carries numerous risks of adverse effects. Increasing or decreasing physical activity is unlikely to influence urinary function.

The nurse observes a client's uric acid level of 9.3 mg/dL. When teaching the client about ways to decrease the uric acid level, which diet would the nurse suggest? a) A diet high in fruits and vegetables b) A low-purine diet c) A diet high in calcium d) A low-sodium diet

A low-purine diet The nurse would suggest a low-purine diet. Foods to avoid are anchovies, animal organs and sardines. The other options do not lower the uric acids levels.

A man tells the nurse that his father died of prostate cancer and he is concerned about his own risk of developing the disease, having heard that prostate cancer has a genetic link. What aspect of the pathophysiology of prostate cancer would underlie the nurses response?

A number of studies have identified an association of BRCA-2 mutation with an increased risk of prostate cancer.

A nurse on a busy medical unit provides care for many patients who require indwelling urinary catheters at some point during their hospital care. The nurse should recognize a heightened risk of injury associated with indwelling catheter use in which patient?

A patient who has Alzheimers disease and who is acutely agitated

A nursing student is learning how to perform sexual assessments using the PLISSIT model. According to this model, the student should begin an assessment by doing which of the following?

Asking the patient if he is willing to discuss sexual functioning

The nurse is caring for a client who is describing urinary symptoms of needing to go to the bathroom with little notice. When the nurse is documenting these symptoms, which medical term will the nurse document? a) Urinary stasis b) Urinary urgency c) Urinary incontinence d) Urinary frequency

Urinary urgency The nurse would document urinary urgency. Urinary frequency is urinating more frequently than normal often times due to inadequate emptying of the bladder. Urinary incontinence is the involuntary loss of urine. Urinary stasis is a stoppage or diminution of flow.

A female patient's most recent urinalysis results are suggestive of bacteriuria. When assessing this patient, the nurse's data analysis should be informed by what principle?

Urine samples are frequently contaminated by bacteria normally present in the urethral area.

A nurse is caring for a female patient whose urinary retention has not responded to conservative treatment. When educating this patient about self-catheterization, the nurse should encourage what practice?

Using clean technique at home to catheterize The patient may use a clean (nonsterile) technique at home, where the risk of cross-contamination is reduced

The nurse recognizes that urinalysis results that most likely indicate a urinary tract infection include: a) proteinuria b) RBC 3 c) WBC 50 d) glucose trace

WBC 50 Increased white blood cell occurs in all clients with a UTI and indicates an infectious process is occurring.

A nurse is teaching a female client with a history of multiple urinary tract infections (UTIs). Which statement indicates the client understands the teaching about preventing UTIs? a) "I should wipe from back to front." b) "I should limit my fluid intake to limit my trips to the bathroom." c) "I should take a tub bath at least 3 times per week." d) "I should take at least 1,000 mg of vitamin C each day."

"I should take at least 1,000 mg of vitamin C each day." The client demonstrates understanding of teaching when she states that she should take vitamin C each day. Increasing vitamin C intake to at least 1,000 mg per day helps acidify the urine, decreasing the amount of bacteria that can grow. The client should wipe from front to back to avoid introducing bacteria from the anal area into the urethra. The client should shower, not bathe, to minimize the amount of bacteria that can enter the urethra. The client should increase her fluid intake, and void every 2 to 3 hours and completely empty her bladder. Holding urine in the bladder can cause the bladder to become distended, which places the client at further risk for UTI.

A client presents at the clinic with complaints of urinary retention. What question should the nurse ask to obtain additional information about the client's complaint? a) "How much fluid are you drinking?" b) "Do you get up at night to urinate?" c) "When did you last urinate?" d) "Have you had a fever and chills?"

"When did you last urinate?" The nurse needs to determine the last time the client voided.

A 32-year-old client has a history of neurogenic bladder and presents with fever, burning, and suprapubic pain. What would you suspect is the problem? a) Urethral strictures b) Urinary incontinence c) Urinary tract infection d) Urinary retention

Urinary tract infection Signs of a bladder infection include fever, chills, and suprapubic pain.

The nurse is caring for a patient recently diagnosed with renal calculi. The nurse should instruct the patient to increase fluid intake to a level where the patient produces at least how much urine each day?

2,000 mL

A patient with cancer of the bladder has just returned to the unit from the PACU after surgery to create an ileal conduit. The nurse is monitoring the patient's urine output hourly and notifies the physician when the hourly output is less than what?

30 mL

The nurse is participating in a bladder retraining program for a patient who had an indwelling catheter for 2 weeks. The nurse knows that, during this process, straight catheterization, after catheter-free intervals, can be discontinued when residual urine is: a) <100 mL b) 400 mL c) 200 mL d) 500 mL

<100 mL Residual urine greater than 100 mL is considered diagnostic of urinary retention. Refer to Box 28-9 in the text.

19. A man tells the nurse that his father died of prostate cancer and he is concerned about his own risk of developing the disease, having heard that prostate cancer has a genetic link. What aspect of the pathophysiology of prostate cancer would underlie the nurse's response? A) A number of studies have identified an association of BRCA-2 mutation with an increased risk of prostate cancer. B) HNPCC is a mutation of two genes that causes prostate cancer in men and it is autosomal dominant. C) Studies have shown that the presence of the TP53 gene strongly influences the incidence of prostate cancer. D) Recent research has demonstrated that prostate cancer is the result of lifestyle factors and that genetics are unrelated.

A Feedback: A number of studies have identified an association of BRCA-2 mutation with an increased risk of prostate cancer. HPNCC is a form of colon cancer. The TP53 gene is associated with breast cancer.

31. A nurse is providing care for a patient who has recently been admitted to the postsurgical unit from PACU following a transuretheral resection of the prostate. The nurse is aware of the nursing diagnosis of Risk for Imbalanced Fluid Volume. In order to assess for this risk, the nurse should prioritize what action? A) Closely monitoring the input and output of the bladder irrigation system B) Administering parenteral nutrition and fluids as ordered C) Monitoring the patient's level of consciousness and skin turgor D) Scanning the patient's bladder for retention every 2 hours

A Feedback: Continuous bladder irrigation effectively reduces the risk of clots in the GU tract but also creates a risk for fluid volume excess if it becomes occluded. The nurse must carefully compare input and output, and ensure that these are in balance. Parenteral nutrition is unnecessary after prostate surgery and skin turgor is not an accurate indicator of fluid status. Frequent bladder scanning is not required when a urinary catheter is in situ.

13. A patient has just returned to the floor following a transurethral resection of the prostate. A triple-lumen indwelling urinary catheter has been inserted for continuous bladder irrigation. What, in addition to balloon inflation, are the functions of the three lumens? A) Continuous inflow and outflow of irrigation solution B) Intermittent inflow and continuous outflow of irrigation solution C) Continuous inflow and intermittent outflow of irrigation solution D) Intermittent flow of irrigation solution and prevention of hemorrhage

A Feedback: For continuous bladder irrigation, a triple-lumen indwelling urinary catheter is inserted. The three lumens provide for balloon inflation and continuous inflow and outflow of irrigation solution.

24. A patient who is postoperative day 12 and recovering at home following a laparoscopic prostatectomy has reported that he is experiencing occasional "dribbling" of urine. How should the nurse best respond to this patient's concern? A) Inform the patient that urinary control is likely to return gradually. B) Arrange for the patient to be assessed by his urologist. C) Facilitate the insertion of an indwelling urinary catheter by the home care nurse. D) Teach the patient to perform intermittent self-catheterization.

A Feedback: It is important that the patient know that regaining urinary control is a gradual process; he may continue to dribble after being discharged from the hospital, but this should gradually diminish (usually within 1 year). At this point, medical follow-up is likely not necessary. There is no need to perform urinary catheterization.

32. A 22-year-old male is being discharged home after surgery for testicular cancer. The patient is scheduled to begin chemotherapy in 2 weeks. The patient tells the nurse that he doesn't think he can take weeks or months of chemotherapy, stating that he has researched the adverse effects online. What is the most appropriate nursing action for this patient at this time? A) Provide empathy and encouragement in an effort to foster a positive outlook. B) Tell the patient it is his decision whether to accept or reject chemotherapy. C) Report the patient's statement to members of his support system. D) Refer the patient to social work.

A Feedback: Patients may be required to endure a long course of therapy and will need encouragement to maintain a positive attitude. It is certainly the patient's ultimate decision to accept or reject chemotherapy, but the nurse should focus on promoting a positive outlook. It would be a violation of confidentiality to report the patient's statement to members of his support system and there is no obvious need for a social work referral.

16. A 35-year-old man is seen in the clinic because he is experiencing recurring episodes of urinary frequency, dysuria, and fever. The nurse should recognize the possibility of what health problem? A) Chronic bacterial prostatitis B) Orchitis C) Benign prostatic hyperplasia D) Urolithiasis

A Feedback: Prostatitis is an inflammation of the prostate gland that is often associated with lower urinary tract symptoms and symptoms of sexual discomfort and dysfunction. Symptoms are usually mild, consisting of frequency, dysuria, and occasionally urethral discharge. Urinary incontinence and retention occur with benign prostatic hyperplasia or hypertrophy. The patient may experience nocturia, urgency, decrease in volume and force of urinary stream. Urolithiasis is characterized by excruciating pain. Orchitis does not cause urinary symptoms.

A nurse is working with a female patient who has developed stress urinary incontinence. Pelvic floor muscle exercises have been prescribed by the primary care provider. How can the nurse best promote successful treatment?

Arrange for biofeedback when the patient is learning to perform the exercises. Biofeedback-assisted pelvic muscle exercise (PME) uses either electromyography or manometry to help the individual identify the pelvic muscles as he or she attempts to learn which muscle group is involved when performing PME

17. To decrease glandular cellular activity and prostate size, an 83-year-old patient has been prescribed finasteride (Proscar). When performing patient education with this patient, the nurse should be sure to tell the patient what? A) Report the planned use of dietary supplements to the physician. B) Decrease the intake of fluids to prevent urinary retention. C) Abstain from sexual activity for 2 weeks following the initiation of treatment. D) Anticipate a temporary worsening of urinary retention before symptoms subside.

A Feedback: Some herbal supplements are contraindicated with Proscar, thus their planned use should be discussed with the physician or pharmacist. The patient should maintain normal fluid intake. There is no need to abstain from sexual activity and a worsening of urinary retention is not anticipated.

12. A public health nurse is teaching a health class for the male students at the local high school. The nurse is teaching the boys to perform monthly testicular self-examinations. What point would be appropriate to emphasize? A) Testicular cancer is a highly curable type of cancer. B) Testicular cancer is very difficult to diagnose. C) Testicular cancer is the number one cause of cancer deaths in males. D) Testicular cancer is more common in older men.

A Feedback: Testicular cancer is highly curable, particularly when it's treated in its early stage. Self-examination allows early detection and facilitates the early initiation of treatment. The highest mortality rates from cancer among men are with lung cancer. Testicular cancer is found more commonly in younger men.

11. A nurse is teaching a 53-year-old man about prostate cancer. What information should the nurse provide to best facilitate the early identification of prostate cancer? A) Have a digital rectal examination and prostate-specific antigen (PSA) test done yearly. B) Have a transrectal ultrasound every 5 years. C) Perform monthly testicular self-examinations, especially after age 60. D) Have a complete blood count (CBC), blood urea nitrogen (BUN) and creatinine assessment performed annually.

A Feedback: The incidence of prostate cancer increases after age 50. The digital rectal examination, which identifies enlargement or irregularity of the prostate, and the PSA test, a tumor marker for prostate cancer, are effective diagnostic measures that should be done yearly. Testicular self-examinations won't identify changes in the prostate gland due to its location in the body. A transrectal ultrasound and CBC with BUN and creatinine assessment are usually done after diagnosis to identify the extent of disease and potential metastases.

38. The nurse is leading a workshop on sexual health for men. The nurse should teach participants that organic causes of erectile dysfunction include what? Select all that apply. A) Diabetes B) Testosterone deficiency C) Anxiety D) Depression E) Parkinsonism

A,B,E Feedback: Organic causes of ED include cardiovascular disease, endocrine disease (diabetes, pituitary tumors, testosterone deficiency, hyperthyroidism, and hypothyroidism), cirrhosis, chronic renal failure, genitourinary conditions (radical pelvic surgery), hematologic conditions (Hodgkin disease, leukemia), neurologic disorders (neuropathies, parkinsonism, spinal cord injury, multiple sclerosis), trauma to the pelvic or genital area, alcohol, smoking, medications, and drug abuse. Anxiety and depression are considered to be psychogenic causes.

22. A 76-year-old with a diagnosis of penile cancer has been admitted to the medical floor. Because the incidence of penile cancer is so low, the staff educator has been asked to teach about penile cancer. What risk factors should the educator cite in this presentation? Select all that apply. A) Phimosis B) Priapism C) Herpes simplex infection D) Increasing age E) Lack of circumcision

A,D,E Feedback: Several risk factors for penile cancer have been identified, including lack of circumcision, poor genital hygiene, phimosis, HPV, smoking, ultraviolet light treatment of psoriasis on the penis, increasing age (two-thirds of cases occur in men older than 65 years of age), lichen sclerosus, and balanitis xerotica obliterans. Priapism and HSV are not known risk factors.

A client comes to the emergency department complaining of severe pain in the right flank, nausea, and vomiting. The physician tentatively diagnoses right ureterolithiasis (renal calculi). When planning this client's care, the nurse should assign the highest priority to which nursing diagnosis? a) Impaired urinary elimination b) Imbalanced nutrition: Less than body requirements c) Acute pain d) Risk for infection

Acute pain Ureterolithiasis typically causes such acute, severe pain that the client can't rest and becomes increasingly anxious. Therefore, the nursing diagnosis of Acute pain takes highest priority. Diagnoses of Risk for infection and Impaired urinary elimination are appropriate when the client's pain is controlled. A diagnosis of Imbalanced nutrition: Less than body requirements isn't pertinent at this time.

A patient has returned to the floor after undergoing a transurethral resection of the prostate (TURP). The patient has a continuous bladder irrigation system in place. The patient tells you he is experiencing bladder spasms and asks what you can do to relieve his discomfort. What is the most appropriate nursing action to relieve the discomfort of the patient?

Administer a smooth-muscle relaxant as ordered. Muscle relaxants may help relieve bladder spasms

A nurse is providing an educational event to a local mens group about prostate cancer. The nurse should cite an increased risk of prostate cancer in what ethnic group?

African Americans

A patient who is scheduled for an open prostatectomy is concerned about the potential effects of the surgery on his sexual function. What aspect of prostate surgery should inform the nurses response?

All prostatectomies carry a risk of nerve damage and consequent erectile dysfunction

The nurse is employed in an urologist office. Which classification of medication is anticipated for clients having difficulty with urinary incontinence? a) Diuretics b) Anticholinergic c) Cholinergic d) Anticonvulsant

Anticholinergic Pharmacologic agents that can improve bladder retention, emptying, and control include anticholinergic drugs. In this classification are medications such as Detrol, Ditropan, and Urecholine. Diuretics eliminate fluid from the body but do not affect the muscles of urinary elimination. Anticonvulsant and cholinergic medications also do not directly help with control.

An ileal conduit is created for a client after a radical cystectomy. Which of the following would the nurse expect to include in the client's plan of care? a) Exercises to promote sphincter control b) Application of an ostomy pouch c) Irrigating the urinary diversion d) Intermittent catheterizations

Application of an ostomy pouch An ileal conduit involves care of a urinary stoma, much like that of a fecal stoma, including the application of an ostomy pouch, skin protection, and stoma care. Intermittent catheterizations and irrigations are appropriate for a continent urinary diverse such as a Kock or Indiana pouch. Exercises to promote sphincter control are appropriate for an ureterosigmoidoscopy.

A client is frustrated and embarrassed by urinary incontinence. Which measure should the nurse include in a bladder retraining program? a) Establishing a predetermined fluid intake pattern for the client b) Assessing present voiding patterns c) Restricting fluid intake to reduce the need to void d) Encouraging the client to increase the time between voidings

Assessing present voiding patterns The guidelines for initiating bladder retraining include assessing the client's present intake patterns, voiding patterns, and reasons for each accidental voiding. Lowering the client's fluid intake won't reduce or prevent incontinence. The client should be encouraged to drink 1.5 to 2 L of water per day. A voiding schedule should be established after assessment.

The nurse has implemented a bladder retraining program for an older adult patient. The nurse places the patient on a timed voiding schedule and performs an ultrasonic bladder scan after each void. The nurse notes that the patient typically has approximately 50 mL of urine remaining in her bladder after voiding. What would be the nurse's best response to this finding?

Avoid further interventions at this time, as this is an acceptable finding. In adults older than 60 years of age, 50 to 100 mL of residual urine may remain after each voiding because of the decreased contractility of the detrusor muscle

The nurse is collaborating with the wound-ostomy-continence (WOC) nurse to teach a patient how to manage her new ileal conduit in the home setting. To prevent leakage or skin breakdown, the nurse should encourage which of the following practices?

Avoid using moisturizing soaps and body washes when cleaning the peristomal area.

28. A patient who is scheduled for an open prostatectomy is concerned about the potential effects of the surgery on his sexual function. What aspect of prostate surgery should inform the nurse's response? A) Erectile dysfunction is common after prostatectomy as a result of hormonal changes. B) All prostatectomies carry a risk of nerve damage and consequent erectile dysfunction. C) Erectile dysfunction after prostatectomy is expected, but normally resolves within several months. D) Modern surgical techniques have eliminated the risk of erectile dysfunction following prostatectomy.

B Feedback: All prostatectomies carry a risk of impotence because of potential damage to the pudendal nerves. If this damage occurs, the effects are permanent. Hormonal changes do not affect sexual functioning after prostatectomy.

40. A patient presents to the emergency department with paraphimosis. The physician is able to compress the glans and manually reduce the edema. Once the inflammation and edema subside, what is usually indicated? A) Needle aspiration of the corpus cavernosum B) Circumcision C) Abstinence from sexual activity for 6 weeks D) Administration of vardenafil

B Feedback: Circumcision is usually indicated after the inflammation and edema subside. Needle aspiration of the corpus cavernosum is indicated in priapism; abstinence from sexual activity for 6 weeks is not indicated. Vardenafil is Levitra and would not be used for paraphimosis.

25. A physician explains to the patient that he has an inflammation of the Cowper glands. Where are the Cowper glands located? A) Within the epididymis B) Below the prostate, within the posterior aspect of the urethra C) On the inner epithelium lining the scrotum, lateral to the testes D) Medial to the vas deferens

B Feedback: Cowper glands lie below the prostate, within the posterior aspect of the urethra. This gland empties its secretions into the urethra during ejaculation, providing lubrication. The Cowper glands do not lie within the epididymis, within the scrotum, or alongside the vas deferens.

15. A man comes to the clinic complaining that he is having difficulty obtaining an erection. When reviewing the patient's history, what might the nurse note that contributes to erectile dysfunction? A) The patient has been treated for a UTI twice in the past year. B) The patient has a history of hypertension. C) The patient is 66 years old. D) The patient leads a sedentary lifestyle.

B Feedback: Past history of infection and lack of exercise do not contribute to impotence. With advancing age, sexual function and libido and potency decrease somewhat, but this is not the primary reason for impotence. Vascular problems cause about half the cases of impotence in men older than 50 years; hypertension is a major cause of such problems.

10. A 55-year-old man presents at the clinic complaining of erectile dysfunction. The patient has a history of diabetes. The physician orders tadalafil (Cialis) to be taken 1 hour before sexual intercourse. The nurse reviews the patient's history prior to instructing the patient on the use of this medication. What disorder will contraindicate the use of tadalafil (Cialis)? A) Cataracts B) Retinopathy C) Hypotension D) Diabetic nephropathy

B Feedback: Patients with cataracts, hypotension, or nephropathy will be allowed to take tadalafil (Cialis) and sildenafil (Viagra) if needed. However, tadalafil (Cialis) and sildenafil (Viagra) are usually contraindicated with diabetic retinopathy.

23. A 75-year-old male patient is being treated for phimosis. When planning this patient's care, what health promotion activity is most directly related to the etiology of the patient's health problem? A) Teaching the patient about safer sexual practices B) Teaching the patient about the importance of hygiene C) Teaching the patient about the safe use of PDE-5 inhibitors D) Teaching the patient to perform testicular self-examination

B Feedback: Poor hygiene often contributes to cases of phimosis. This health problem is unrelated to sexual practices, the use of PDE-5 inhibitors, or testicular self-examination.

21. A 35-year-old father of three tells the nurse that he wants information on a vasectomy. What would the nurse tell him about ejaculate after a vasectomy? A) There will be no ejaculate after a vasectomy, though the patient's potential for orgasm is unaffected. B) There is no noticeable decrease in the amount of ejaculate even though it contains no sperm. C) There is a marked decrease in the amount of ejaculate after vasectomy, though this does not affect sexual satisfaction. D) There is no change in the quantity of ejaculate after vasectomy, but the viscosity is somewhat increased.

B Feedback: Seminal fluid is manufactured predominantly in the seminal vesicles and prostate gland, which are unaffected by vasectomy, thus no noticeable decrease in the amount of ejaculate occurs (volume decreases approximately 3%), even though it contains no spermatozoa. The viscosity of ejaculate does not change.

4. A patient has been prescribed sildenafil. What should the nurse teach the patient about this medication? A) Sexual stimulation is not needed to obtain an erection. B) The drug should be taken 1 hour prior to intercourse. C) Facial flushing or headache should be reported to the physician immediately. D) The drug has the potential to cause permanent visual changes.

B Feedback: The patient must have sexual stimulation to create the erection, and the drug should be taken 1 hour before intercourse. Facial flushing, mild headache, indigestion, and running nose are common side effects of Viagra and do not normally warrant reporting to the physician. Some visual disturbances may occur, but these are transient.

35. A clinic nurse is providing preprocedure education for a man who will undergo a vasectomy. Which of the following measures will enhance healing and comfort? Select all that apply. A) Abstaining from sexual intercourse for at least 14 days postprocedure B) Wearing a scrotal support garment C) Using sitz baths D) Applying a heating pad intermittently E) Staying on bed rest for 48 to 72 hours postprocedure

B,C Feedback: Applying ice bags intermittently to the scrotum for several hours after surgery can reduce swelling and relieve discomfort, and is preferable to the application of heat. The nurse advises the patient to wear snug, cotton underwear or a scrotal support for added comfort and support. Sitz baths can also enhance comfort. Extended bed rest is unnecessary, and sexual activity can usually be resumed in 1 week.

A 29-year-old patient has just been told that he has testicular cancer and needs to have surgery. During a presurgical appointment, the patient admits to feeling devastated that he requires surgery, stating that it will leave him emasculated and a shell of a man. The nurse should identify what nursing diagnosis when planning the patients subsequent care?

Disturbed Body Image Related to Effects of Surgery The patients statements specifically address his perception of his body as it relates to his identity

A client is suspected of having interstitial cystitis. Which diagnostic test would the nurse anticipate as being used to confirm the diagnosis? a) Bladder biopsy b) Voiding cystourethrogram c) Urine culture d) Cystoscopy

Bladder biopsy A biopsy of the bladder mucosa which reveals an inflammatory process with scarring and hemorrhagic areas confirms the diagnosis. A cystoscopy would reveal a markedly inflamed bladder with pinpoint hemorrhage and a bladder capacity that is smaller than normal. A voiding cystourethrogram demonstrates a small bladder capacity. Urine culture would be negative.

Behavioral interventions for urinary incontinence can be coordinated by a nurse. A comprehensive program that incorporates timed voiding and urinary urge inhibition is referred to as: a) Voiding at given intervals. b) Bladder retaining c) Interval voiding d) Prompted voiding

Bladder retaining Bladder retraining includes a timed voiding schedule and urinary urge inhibition exercises. These exercises involve delaying voiding to help the patient stay dry for a set period of time. When one time interval is reached, another is set. The time is usually increased by 10 to 15 minutes, until an acceptable voiding interval is achieved.

1. An adolescent is identified as having a collection of fluid in the tunica vaginalis of his testes. The nurse knows that this adolescent will receive what medical diagnosis? A) Cryptorchidism B) Orchitis C) Hydrocele D) Prostatism

C Feedback: A hydrocele refers to a collection of fluid in the tunica vaginalis of the testes. Cryptorchidism is the most common congenital defect in males, characterized by failure of one or both of the testes to descend into the scrotum. Orchitis is an inflammation of the testes (testicular congestion) caused by pyogenic, viral, spirochetal, parasitic, traumatic, chemical, or unknown factors. Prostatism is an obstructive and irritative symptom complex that includes increased frequency and hesitancy in starting urination, a decrease in the volume and force of the urinary stream, acute urinary retention, and recurrent urinary tract infections.

20. A nurse is performing an admission assessment on a 40-year-old man who has been admitted for outpatient surgery on his right knee. While taking the patient's family history, he states, "My father died of prostate cancer at age 48." The nurse should instruct him on which of the following health promotion activities? A) The patient will need PSA levels drawn starting at age 55. B) The patient should have testing for presence of the CDH1 and STK11 genes. C) The patient should have PSA levels drawn regularly. D) The patient should limit alcohol use due to the risk of malignancy.

C Feedback: PSA screening is warranted by the patient's family history and should not be delayed until age 55. The CDH1 and STK11 genes do not relate to the risk for prostate cancer. Alcohol consumption by the patient should be limited. However, this is not the most important health promotion intervention.

3. A nurse practitioner is assessing a 55-year-old male patient who is complaining of perineal discomfort, burning, urgency, and frequency with urination. The patient states that he has pain with ejaculation. The nurse knows that the patient is exhibiting symptoms of what? A) Varicocele B) Epididymitis C) Prostatitis D) Hydrocele

C Feedback: Perineal discomfort, burning, urgency, frequency with urination, and pain with ejaculation is indicative of prostatitis. A varicocele is an abnormal dilation of the pampiniform venous plexus and the internal spermatic vein in the scrotum (the network of veins from the testis and the epididymis that constitute part of the spermatic cord). Epididymitis is an infection of the epididymis that usually descends from an infected prostate or urinary tract; it also may develop as a complication of gonorrhea. A hydrocele is a collection of fluid, generally in the tunica vaginalis of the testis, although it also may collect within the spermatic cord.

27. A nurse is caring for a 33-year-old male who has come to the clinic for a physical examination. He states that he has not had a routine physical in 5 years. During the examination, the physician finds that digital rectal examination (DRE) reveals "stoney" hardening in the posterior lobe of the prostate gland that is not mobile. The nurse recognizes that the observation typically indicates what? A) A normal finding B) A sign of early prostate cancer C) Evidence of a more advanced lesion D) Metastatic disease

C Feedback: Routine repeated DRE (preferably by the same examiner) is important, because early cancer may be detected as a nodule within the gland or as an extensive hardening in the posterior lobe. The more advanced lesion is stony hard and fixed. This finding is not suggestive of metastatic disease.

26. A nursing student is learning how to perform sexual assessments using the PLISSIT model. According to this model, the student should begin an assessment by doing which of the following? A)Briefly teaching the patient about normal sexual physiology B)Assuring the patient that what he says will be confidential C)Asking the patient if he is willing to discuss sexual functioning D)Ensuring patient privacy

C Feedback: The PLISSIT (permission, limited information, specific suggestions, intensive therapy) model of sexual assessment and intervention may be used to provide a framework for nursing interventions. By beginning with the patient's permission, the nurse establishes a patient-centered focus.

14. A nurse is assessing a patient who presented to the ED with priapism. The student nurse is aware that this condition is classified as a urologic emergency because of the potential for what? A) Urinary tract infection B) Chronic pain C) Permanent vascular damage D) Future erectile dysfunction

C Feedback: The ischemic form of priapism, which is described as nonsexual, persistent erection with little or no cavernous blood flow, must be treated promptly to prevent permanent damage to the penis. Priapism has not been indicated in the development of UTIs, chronic pain, or erectile dysfunction.

The nurse is caring for a patient with an indwelling urinary catheter. The nurse is aware that what nursing action helps prevent infection in a patient with an indwelling catheter?

Empty the drainage bag at least every 8 hours.

The nurse working with a patient after an ileal conduit notices that the pouching system is leaking small amounts of urine. The appropriate nursing intervention is which of the following? a) Empty the pouch. b) Secure/patch it with tape. c) Secure/patch with barrier paste. d) Change wafer and pouch.

Change wafer and pouch. Whenever the nurse notes a leaking pouching system, the nurse should change the wafer and pouch. Attempting to secure or patch the leak with tape and/or barrier paste will trap urine under the barrier or faceplate, which will compromise peristomal skin integrity. Emptying the pouch will not rectify the leaking.

A 35-year-old man is seen in the clinic because he is experiencing recurring episodes of urinary frequency, dysuria, and fever. The nurse should recognize the possibility of what health problem?

Chronic bacterial prostatitis Prostatitis is an inflammation of the prostate gland that is often associated with lower urinary tract symptoms and symptoms of sexual discomfort and dysfunction. Symptoms are usually mild, consisting of frequency, dysuria, and occasionally urethral discharge.

A patient has been diagnosed with a UTI and is prescribed an antibiotic. What first-line fluoroquinolone antibacterial agent for UTIs has been found to be significantly effective? a) Cipro b) Bactrim c) Macrodantin d) Septra

Cipro Ciprofloxacin (Cipro) is a fluoroquinolone used to treat UTIs. Co-trimoxazole (Bactrim, Septra) is a trimethoprim-sulfamethoxazole combination medication. Nitrofurantoin (Macrodantin, Furadantin) is an anti-infective urinary tract medication.

The nurse is caring for an older patient whose chart reveals that the patient has a reversible cause of urinary incontinence. The nurse creates a plan of care for which of the following conditions? a) Constipation b) Asthma c) Bladder cancer d) Decreased progesterone levels

Constipation Constipation is a reversible cause of urinary incontinence in the older adult. Other reversible causes include acute urinary tract infection, infection elsewhere in the body, decreased fluid intake, a change in a chronic disease pattern, and decreased estrogen levels in the menopausal woman. The other answers do not apply.

The nurse, in assessing a patient's newly created stoma, observes that the stoma color is now dark purple. The appropriate nursing intervention is to do which of the following? a) Remove the urinary stents. b) Apply Karaya powder. c) Change the pouching system. d) Contact the physician.

Contact the physician. The appropriate nursing intervention when a newly created stoma is dark purple is to notify the physician. The physician or wound, ostomy, and continence (WOC) nurse will assess the stoma to determine if it the stoma has superficial ischemia or if it is necrotic.

A patient has just returned to the floor following a transurethral resection of the prostate. A triple-lumen indwelling urinary catheter has been inserted for continuous bladder irrigation. What, in addition to balloon inflation, are the functions of the three lumens?

Continuous inflow and outflow of irrigation solution

36. A patient has returned to the floor after undergoing a transurethral resection of the prostate (TURP). The patient has a continuous bladder irrigation system in place. The patient tells you he is experiencing bladder spasms and asks what you can do to relieve his discomfort. What is the most appropriate nursing action to relieve the discomfort of the patient? A) Apply a cold compress to the pubic area. B) Notify the urologist promptly. C) Irrigate the catheter with 30 to 50 mL of normal saline aS ordered. D) Administer a smooth-muscle relaxant as ordered.

D Feedback: Administering a medication that relaxes smooth muscles can help relieve bladder spasms. Neither a cold compress nor catheter irrigation will alleviate bladder spasms. In most cases, this problem can be relieved without the involvement of the urologist, who will normally order medications on a PRN basis.

8. A patient has just been diagnosed with prostate cancer and is scheduled for brachytherapy next week. The patient and his wife are unsure of having the procedure because their daughter is 3 months pregnant. What is the most appropriate teaching the nurse should provide to this family? A) The patient should not be in contact with the baby after delivery. B) The patient's treatment poses no risk to his daughter or her infant. C) The patient's brachytherapy may be contraindicated for safety reasons. D) The patient should avoid close contact with his daughter for 2 months.

D Feedback: Brachytherapy involves the implantation of interstitial radioactive seeds under anesthesia. The surgeon uses ultrasound guidance to place about 80 to 100 seeds, and the patient returns home after the procedure. Exposure of others to radiation is minimal, but the patient should avoid close contact with pregnant women and infants for up to 2 months.

7. A nurse is planning the postoperative care of a patient who is scheduled for radical prostatectomy. What intraoperative position will place the patient at particular risk for the development of deep vein thrombosis postoperatively? A) Fowler's position B) Prone position C) Supine position D) Lithotomy position

D Feedback: Elastic compression stockings are applied before surgery and are particularly important for prevention of deep vein thrombosis if the patient is placed in a lithotomy position during surgery. During a prostatectomy, the patient is not placed in the supine, prone, or Fowler's position.

9. A patient has presented at the clinic with symptoms of benign prostatic hyperplasia. What diagnostic findings would suggest that this patient has chronic urinary retention? A) Hypertension B) Peripheral edema C) Tachycardia and other dysrhythmias D) Increased blood urea nitrogen (BUN)

D Feedback: Hypertension, edema, and tachycardia would not normally be associated with benign prostatic hyperplasia. Azotemia is an accumulation of nitrogenous waste products, and renal failure can occur with chronic urinary retention and large residual volumes.

37. A patient confides to the nurse that he cannot engage in sexual activity. The patient is 27 years old and has no apparent history of chronic illness that would contribute to erectile dysfunction. What does the nurse know will be ordered for this patient to assess his sexual functioning? A) Sperm count B) Ejaculation capacity tests C) Engorgement tests D) Nocturnal penile tumescence tests

D Feedback: Nocturnal penile tumescence tests may be conducted in a sleep laboratory to monitor changes in penile circumference during sleep using various methods to determine number, duration, rigidity, and circumference of penile erections; the results help identify whether the erectile dysfunction is caused by physiologic and/or psychological factors. A sperm count would be done if the patient was complaining of infertility. Ejaculation capacity tests and engorgement tests are not applicable for assessment in this circumstance.

39. A patient has been diagnosed with erectile dysfunction; the cause has been determined to be psychogenic. The patient's interdisciplinary plan of care should prioritize which of the following interventions? A) Penile implant B) PDE-5 inhibitors C) Physical therapy D) Psychotherapy

D Feedback: Patients with erectile dysfunction from psychogenic causes are referred to a health care provider or therapist who specializes in sexual dysfunction. Because of the absence of an organic cause, medications and penile implants are not first-line treatments. Physical therapy is not normally effective in the treatment of ED.

33. A 57-year-old male comes to the clinic complaining that when he has an erection his penis curves and becomes painful. The patient's diagnosis is identified as severe Peyronie's disease. The nurse should be aware of what likely treatment modality? A) Physical therapy B) Treatment with PDE-5 inhibitors C) Intracapsular hydrocortisone injections D) Surgery

D Feedback: Surgical removal of mature plaques is used to treat severe Peyronie's disease. There is no potential benefit to physical therapy and hydrocortisone injections are not normally used. PDE-5 inhibitors would exacerbate the problem.

6. A public health nurse has been asked to provide a health promotion session for men at a wellness center. What should the nurse inform the participants about testicular cancer? A) It is most common among men over 55. B) It is one of the least curable solid tumors. C) It typically does not metastasize. D) It is highly responsive to treatment.

D Feedback: Testicular cancer is most common among men 15 to 35 years of age and produces a painless enlargement of the testicle. Testicular cancers metastasize early but are one of the most curable solid tumors, being highly responsive to chemotherapy.

An adult patient has been hospitalized with pyelonephritis. The nurses review of the patients intake and output records reveals that the patient has been consuming between 3 L and 3.5 L of oral fluid each day since admission. How should the nurse best respond to this finding?

Encourage the patient to continue this pattern of fluid intake

5. A patient is 24 hours postoperative following prostatectomy and the urologist has ordered continuous bladder irrigation. What color of output should the nurse expect to find in the drainage bag? A) Red wine colored B) Tea colored C) Amber D) Light pink

D Feedback: The urine drainage following prostatectomy usually begins as a reddish pink, then clears to a light pink 24 hours after surgery.

A gerontologic nurse is assessing a patient who has numerous comorbid health problems. What assessment findings should prompt the nurse to suspect a UTI? Select all that apply. A) Food cravings B) Upper abdominal pain C) Insatiable thirst D) Uncharacteristic fatigue E) New onset of confusion

D) Uncharacteristic fatigue - this answer only, even with Select All That Apply!! The most common subjective presenting symptom of UTI in older adults is generalized fatigue. The most common objective finding is a change in cognitive functioning.

After teaching a group of students about the types of urinary incontinence and possible causes, the instructor determines that the student have understood the material when they identify which of the following as a cause of stress incontinence? a) Obstruction due to fecal impaction or enlarged prostate b) Increased urine production due to metabolic conditions c) Decreased pelvic muscle tone due to multiple pregnancies d) Bladder irritation related to urinary tract infections

Decreased pelvic muscle tone due to multiple pregnancies Stress incontinence is due to decreased pelvic muscle tone, which is associated with multiple pregnancies, obstetric injuries, obesity, menopause, or pelvic disease. Transient incontinence is due to increased urine production related to metabolic conditions. Urge incontinence is due to bladder irritation related to urinary tract infections, bladder tumors, radiation therapy, enlarged prostate, or neurologic dysfunction. Overflow incontinence is due to obstruction from fecal impaction or enlarged prostate.

The nurse is conducting a community education program on UTIs. The nurse determines that the participants understand the teaching when they identify which of the following as a contributing factor for UTIs in older adults? a) Sporadic use of antimicrobial agents b) Active lifestyle c) Immunocompromise d) Low incidence of chronic illness

Immunocompromise Factors that contribute to UTIs in older adults include immunocompromise, high incidence of chronic illness, immobility, and frequent use of antimicrobial agents.

Which of the following is a factor contributing to UTI in older adults? a) Immunocompromise b) Active lifestyle c) Sporadic use of antimicrobial agents d) Low incidence of chronic illness

Immunocompromise Factors that contribute to urinary tract infection in older adults include immunocompromise, high incidence of chronic illness, immobility, and frequent use of antimicrobial agents.

The nurse is assisting in the development of a protocol for bladder retraining following removal of an indwelling catheter. Which of the following should the nurse include? a) Implementing a 2- to 3-hour voiding schedule b) Avoiding drinking fluids for 6 hours c) Performing straight catheterization every 4 hours d) Encouraging voiding immediately after catheter removal

Implementing a 2- to 3-hour voiding schedule Immediately after the removal of the indwelling catheter, the patient is placed on a voiding schedule, usually 2 to 3 hours. At the given time interval, the patient is instructed to void. If bladder ultrasound scanning shows 100 mL or more of urine remaining in the bladder after voiding, straight catheterization may be performed for complete bladder emptying. Immediate voiding is not usually encouraged.

The nurse is educating a patient with urolithiasis about preventative measures to avoid another occurrence. What should the patient be encouraged to do? a) Add calcium supplements to the diet to replace losses to renal calculi. b) Participate in strenuous exercises so that the tone of smooth muscle in the urinary tract can be strengthened to help propel calculi. c) Limit voiding to every 6 to 8 hours so that increased volume can increase hydrostatic pressure, which will help push stones along the urinary system. d) Increase fluid intake so that the patient can excrete 2,500 to 4,000 mL every day, which will help prevent additional stone formation.

Increase fluid intake so that the patient can excrete 2,500 to 4,000 mL every day, which will help prevent additional stone formation. A patient who has shown a tendency to form stones should drink enough fluid to excrete greater than 2,000 mL (preferably 3,000 to 4,000 mL) of urine every 24 hours (Meschi et al., 2011).

A patient has been prescribed sildenafil. What should the nurse teach the patient about this medication?

The drug should be taken 1 hour prior to intercourse. The patient must have sexual stimulation to create the erection, and the drug should be taken 1 hour before intercourse. Facial flushing, mild headache, indigestion, and running nose are common side effects of Viagra and do not normally warrant reporting to the physician

The nurse advises the patient with chronic pyelonephritis that he should: a) Decrease his sodium intake to prevent fluid retention. b) Decrease his intake of calcium rich foods to prevent kidney stones. c) Limit his fluid intake to 1.5 L/day to minimize bladder fullness, which could cause backward pressure on the kidneys. d) Increase fluids to 3 to 4 L/24 hours to dilute the urine.

Increase fluids to 3 to 4 L/24 hours to dilute the urine. Unless contraindicated, fluids should be increased to dilute the urine, decrease burning on urination, and prevent dehydration. A balanced diet would be recommended but there is no need to restrict sodium or calcium.

A patient has presented at the clinic with symptoms of benign prostatic hyperplasia. What diagnostic findings would suggest that this patient has chronic urinary retention?

Increased blood urea nitrogen (BUN)

A client is prescribed amitriptyline, an antidepressant for incontinence. The nurse understands that this drug is an effective treatment because it: a) Increases bladder neck resistance. b) Decreases involuntary bladder contractions. c) Reduces bladder spasticity. d) Increases contraction of the detrusor muscle.

Increases bladder neck resistance. Some tricyclic antidepressant medications (amitriptyline, nortriptyline, and amoxapine) are useful in treating incontinence because they decrease bladder contractions and increase bladder neck resistance. Anticholinergic drugs such as oxybutynin chloride (Ditropan), reduce bladder spasticity and involuntary bladder contractions. Bethanechol (Urecholine) helps to increase contraction of the detrusor muscle, which assists with emptying of the bladder.

A patient has been successfully treated for kidney stones and is preparing for discharge. The nurse recognizes the risk of recurrence and has planned the patients discharge education accordingly. What preventative measure should the nurse encourage the patient to adopt?

Increasing fluid intake

A nurse is caring for a client with acute pyelonephritis. Which nursing intervention is the most important? a) Encouraging the client to drink cranberry juice to acidify the urine b) Administering a sitz bath twice per day c) Using an indwelling urinary catheter to measure urine output accurately d) Increasing fluid intake to 3 L/day

Increasing fluid intake to 3 L/day Acute pyelonephritis is a sudden inflammation of the interstitial tissue and renal pelvis of one or both kidneys. Infecting bacteria are normal intestinal and fecal flora that grow readily in urine. Pyelonephritis may result from procedures that involve the use of instruments (such as catheterization, cystoscopy, and urologic surgery) or from hematogenic infection. The most important nursing intervention is to increase fluid intake to 3 L/day. Doing so helps empty the bladder of contaminated urine and prevents calculus formation. Administering a sitz bath would increase the likelihood of fecal contamination. Using an indwelling urinary catheter could cause further contamination. Encouraging the client to drink cranberry juice to acidify urine is helpful but isn't the most important intervention.

The nurse is caring for a patient who has undergone creation of a urinary diversion. Forty-eight hours postoperatively, the nurses assessment reveals that the stoma is a dark purplish color. What is the nurses most appropriate response?

Inform the primary care provider that the vascular supply may be compromised

A patient has returned to the floor from the PACU after undergoing a suprapubic prostatectomy. The nurse notes significant urine leakage around the suprapubic tube. What is the nurses most appropriate action?

Inform the urologist of this finding.

The nurse and urologist have both been unsuccessful in catheterizing a patient with a prostatic obstruction and a full bladder. What approach does the nurse anticipate the physician using to drain the patients bladder?

Insertion of a suprapubic catheter

Patricia O'Connor, a 17-year-old high school student, is returning to the medical-surgical unit where you practice nursing from surgery. She has just undergone an appendectomy. She reports the need to urinate and cannot do so. What is your response to her situation as ordered by the physician? a) Intermittent catheterization b) Clean intermittent catheterization c) Indwelling catheterization d) All options are correct.

Intermittent catheterization Acute retention that is likely to resolve quickly (e.g., after anesthesia) probably will be treated by intermittent catheterization.

A public health nurse has been asked to provide a health promotion session for men at a wellness center. What should the nurse inform the participants about testicular cancer?

It is highly responsive to treatment Testicular cancer is most common among men 15 to 35 years of age and produces a painless enlargement of the testicle

A nurse who is taking care of a patient with a spinal cord injury documents the frequency of reflex incontinence. The nurse understands that this is most likely due to: a) A stricture or tumor in the bladder. b) Loss of motor control of the detrusor muscle. c) Compromised ligament and pelvic floor support of the urethra. d) Uninhibited detrusor contractions.

Loss of motor control of the detrusor muscle. Spinal cord injury patients commonly experience reflex incontinence because they lack neurologically mediated motor control of the detrusor and the sensory awareness of the urge to void. These patients also experience hyperreflexia in the absence of normal sensations associated with voiding.

Examination of a client's bladder stones reveal that they are primarily composed of uric acid. The nurse would expect to provide the client with which type of diet? a) High protein b) Low oxalate c) Low purine d) High sodium

Low purine A low-purine diet is used for uric acid stones, although the benefits are unknown. Clients with a history of calcium oxalate stone formation need a diet that is adequate in calcium and low in oxalate. Only clients who have type II absorptive hypercalciuria—approximately half of the clients—need to limit calcium intake. Usually, clients are told to increase their fluid intake significantly, consume a moderate protein intake, and limit sodium. Avoiding excessive protein intake is associated with lower urinary oxalate and lower uric acid levels. Reducing sodium intake can lower urinary calcium levels.

A patient who has been treated with uric acid for stones is being discharged from the hospital. What type of diet does the nurse discuss with the patient? a) Low-calcium diet b) Low-phosphorus diet c) Low-purine diet d) High-protein diet

Low-purine diet For uric acid stones, the patient is placed on a low-purine diet to reduce the excretion of uric acid in the urine. Foods high in purine (shellfish, anchovies, asparagus, mushrooms, and organ meats) are avoided, and other proteins may be limited.

A patient undergoes surgery for removing a malignant tumor, followed by a urinary diversion procedure. The nurse's postoperative plan of care should include which of the following? a) Determine the patient's ability to manage stoma care. b) Suggest a visit to a local ostomy group. c) Maintain skin and stomal integrity. d) Show pictures and drawings of placement of the stoma.

Maintain skin and stomal integrity. The most important postoperative nursing management is to maintain skin and stomal integrity to avoid further complications, such as skin infections and urinary odor. Determining the patient's ability to manage stoma care, showing photographs, and suggesting a visit to a local ostomy group would be a part of the preoperative procedure.

The nurse is conducting a history and assessment related to a patient's incontinence. Which of the following should the nurse include in the assessment before beginning a bladder training program? a) History of allergies b) Occupational history c) Medication usage d) Smoking habits

Medication usage It is essential to assess the patient's physical and environmental conditions before beginning a bladder training program, because the patient may not be able to reach the bathroom in time. During the bladder training program, a change in environment may be an effective suggestion for the patient. It is not so essential to assess the patient's history of allergy, occupation, and smoking habits before beginning a bladder training program.

A nurse is reviewing the history and physical examination of a client with a suspected malignant tumor of the bladder. Which finding would the nurse identify as the most common initial symptom? a) Urinary retention b) Painless hematuria c) Frequency d) Fever

Painless hematuria The most common first symptom of a malignant tumor of the bladder is painless hematuria. Additional early symptoms include UTI with symptoms such as fever, dysuria, urgency, and frequency. Later symptoms are related to metastases and include pelvic pain, urinary retention (if the tumor blocks the bladder outlet), and urinary frequency from the tumor occupying bladder space.

Ms. Simpson, age 72 years, is being seen in the clinic with a suspected bladder tumor. These tumors occur more frequently in men than women and usually affect clients 50 years of age and older. Use of tobacco products is the leading cause of bladder cancer. You are asking Ms. Simpson about symptoms that she has had that brought her to the clinic. What is the most common first symptom of a malignant tumor of the bladder? a) Urgency b) Fever c) Painless hematuria d) Dysuria

Painless hematuria The most common first symptom of a malignant tumor of the bladder is painless hematuria. Additional early symptoms include UTI with symptoms such as fever, dysuria, urgency, and frequency. Painless hematuria is the most common, however.

A 64-year-old man is seeing his urologist for an annual check-up, post prostatectomy. The health care provider is concerned with the symptom he finds because it is considered diagnostic for bladder cancer. Which of the following signs/symptoms is diagnostic for bladder cancer? a) Deep flank and abdominal pain b) Muscle spasm and abdominal rigidity over the flank c) Painless, gross hematuria d) Decreasing kidney function associated with fever and hematuria

Painless, gross hematuria Although flank pain may occur, the painless, gross hematuria is characteristic of bladder cancer.

A nurse is assessing a patient who presented to the ED with priapism. The student nurse is aware that this condition is classified as a urologic emergency because of the potential for what?

Permanent vascular damage Priapism is described as nonsexual, persistent erection with little or no cavernous blood flow, must be treated promptly to prevent permanent damage to the penis.

Which of the following medications may be ordered to relieve discomfort associated with a UTI? a) Nitrofurantoin (Furadantin) b) Levofloxacin (Levaquin) c) Phenazopyridine (Pyridium) d) Ciprofloxacin (Cipro)

Phenazopyridine (Pyridium) Pyridium is a urinary analgesic ordered to relieve discomfort associated with UTIs. Furadantin, Cipro, and Levaquin are antibiotics.

An uncircumcised 78-year-old male has presented at the clinic complaining that he cannot retract his foreskin over his glans. On examination, it is noted that the foreskin is very constricted. The nurse should recognize the presence of what health problem?

Phimosis Phimosis is the term used to describe a condition in which the foreskin is constricted so that it cannot be retracted over the glans

A patient being treated in the hospital has been experiencing occasional urinary retention. What nursing action should the nurse take to encourage a patient who is having difficulty voiding?

Provide privacy for the patient

A patient has been diagnosed with erectile dysfunction; the cause has been determined to be psychogenic. The patients interdisciplinary plan of care should prioritize which of the following interventions?

Psychotherapy Patients with erectile dysfunction from psychogenic causes are referred to a health care provider or therapist who specializes in sexual dysfunction

A patient is postoperative day 3 following the creation of an ileal conduit for the treatment of invasive bladder cancer. The patient is quickly learning to self-manage the urinary diversion, but expresses concern about the presence of mucus in the urine. What is the nurses most appropriate response?

Reassure the patient that this is an expected phenomenon

A female patient visits her primary health care provider with a complaint of frequency of urination and incontinence when she sneezes. The health care provider suspects the patient is experiencing cystitis. The nurse knows that this is most likely due to which of the following? a) Disturbance in the normal bacterial flora of the vagina b) Dysfunction of the bladder neck or urethra. c) Reflux of urine from the urethra into the bladder d) Interruption in the protective effect of glycosaminoglycan

Reflux of urine from the urethra into the bladder With coughing, sneezing, or straining, the bladder pressure increases, which may force urine from the bladder into the urethra. When the pressure returns to normal, the urine flows back into the bladder, bringing into the bladder bacteria from the anterior portions of the urethra. See Figure 28-1 in the text.

Sympathomimetics have which of the following effects on the body? a) Constriction of pupils b) Decrease of heart rate c) Relaxation of bladder wall d) Constriction of bronchioles

Relaxation of bladder wall Sympathomimetics mimic the sympathetic nervous system, causing increased heart rate and contractility, dilation of bronchioles and pupils, and bladder wall relaxation.

The nurse on a urology unit is working with a patient who has been diagnosed with oxalate renal calculi. When planning this patient's health education, what nutritional guidelines should the nurse provide?

Restrict protein intake as ordered Protein is restricted to 60 g/d, while sodium is restricted to 3 to 4 g/d.

A 55-year-old man presents at the clinic complaining of erectile dysfunction. The patient has a history of diabetes. The physician orders tadalafil (Cialis) to be taken 1 hour before sexual intercourse. The nurse reviews the patients history prior to instructing the patient on the use of this medication. What disorder will contraindicate the use of tadalafil (Cialis)?

Retinopathy tadalafil (Cialis) and sildenafil (Viagra) are usually contraindicated with diabetic retinopathy

An older adult has experienced a new onset of urinary incontinence and family members identify this problem as being unprecedented. When assessing the patient for factors that may have contributed to incontinence, the nurse should prioritize what assessment?

Reviewing the patient's medication administration record for recent changes

Following percutaneous nephrolithotomy, the client is at greatest risk for which nursing diagnosis? a) Risk for fluid volume excess b) Risk for deficient knowledge: self-catherization c) Risk for altered urinary elimination d) Risk for infection

Risk for infection Percutaneous nephrolithotomy is an invasive procedure for the removal of renal calculi. The client would be at risk for infection.

A patient with a sacral pressure ulcer has had a urinary catheter inserted. As a result of this new intervention, the nurse should prioritize what nursing diagnosis in the patients plan of care?

Risk for infection related to presence of an indwelling urinary catheter

The nurse is conducting a community education program on urinary incontinence. The nurse determines that the participants understand the teaching when they identify which of the following as risk factors for urinary incontinence? a) Cesarean delivery b) Sedatives c) Body mass index (BMI) of 22 d) Swimming

Sedatives Use of sedatives, diuretics, hypnotics, and opioids are risk factors for urinary incontinence. Additional risk factors include high-impact exercises, a BMI greater than 40, and vaginal birth delivery.

The clinic nurse is preparing a plan of care for a patient with a history of stress incontinence. What role will the nurse have in implementing a behavioral therapy approach?

Teach the patient to perform pelvic floor muscle exercises. Kegel exercises address stress, urge, and mix incontinence

The nurse is assisting in the transport of a client with an indwelling catheter to the diagnostic studies unit. Which action, made by the nursing assistant, would require instruction? a) The nursing assistant places the drainage bag on the lower area of the wheelchair for transport. b) The nursing assistant places the drainage bag on the client's abdomen for transport. c) The nursing assistant holds the drainage bag while the client moves to the wheelchair. d) The nursing assistant keeps the catheter and drainage bag together when moving the client.

The nursing assistant places the drainage bag on the client's abdomen for transport. The nurse would instruct the nursing assistant to maintain the drainage bag lower than the genital region to avoid a backflow of urine into the bladder. The nursing assistant is correct to move the catheter and drainage bag with the client to not put tension on the catheter, place the drainage bag on the lower area of the wheelchair, and hold the drainage bag while the client is in the process of moving.

A man comes to the clinic complaining that he is having difficulty obtaining an erection. When reviewing the patients history, what might the nurse note that contributes to erectile dysfunction?

The patient has a history of hypertension

A patient has just been diagnosed with prostate cancer and is scheduled for brachytherapy next week. The patient and his wife are unsure of having the procedure because their daughter is 3 months pregnant. What is the most appropriate teaching the nurse should provide to this family?

The patient should avoid close contact with his daughter for 2 months Exposure of others to radiation is minimal, but the patient should avoid close contact with pregnant women and infants for up to 2 months.

A nurse is performing an admission assessment on a 40-year-old man who has been admitted for outpatient surgery on his right knee. While taking the patients family history, he states, My father died of prostate cancer at age 48. The nurse should instruct him on which of the following health promotion activities?

The patient should have PSA levels drawn regularly PSA screening is warranted by the patients family history and should not be delayed until age 55

The nurse is working with a patient who has been experiencing episodes of urinary retention. What assessment finding would suggest that the patient is experiencing retention?

The patient's suprapubic region is dull on percussion

The nurse is teaching a health class about UTIs to a group of older adults. What characteristic of UTIs should the nurse cite?

The prevalence of UTIs in men older than 50 years of age approaches that of women in the same age group.

A female patient has been prescribed a course of antibiotics for the treatment of a UTI. When providing health education for the patient, the nurse should address what topic?

The risk of developing a vaginal yeast infection as a consequent of antibiotic therapy

The nurse is caring for a client with recurrent urinary tract infections. Which of the following body structures would the nurse instruct as the most frequent cause of women's urinary tract infections? a) The ureters b) The rectum c) The urethra d) The bladder

The urethra Because the urethra is short in women, ascending infections or microorganisms carried from the vagina or rectum are common. Males have a longer urethra, causing the organisms travel farther to the bladder. Although structures of the urinary system, the other options are where the client has bacteria and microorganisms located. The ureters connect the bladder to kidney thus do not obtain bacteria, just transmit when available.

A patient has been admitted to the postsurgical unit following the creation of an ileal conduit. What should the nurse measure to determine the size of the appliance needed?

The widest part of the stoma

A 35-year-old father of three tells the nurse that he wants information on a vasectomy. What would the nurse tell him about ejaculate after a vasectomy?

There is no noticeable decrease in the amount of ejaculate even though it contains no sperm.

James Roth, a 63-year-old accountant, is a client on the hospital unit where you practice nursing. Mr. Roth has developed urinary incontinence and is beginning bladder training to regain control over his urine elimination. Why is the catheter being clamped and unclamped? a) To prevent bladder distention b) To promote normal bladder function c) To prevent urinary retention d) To promote urine production

To promote normal bladder function The clamping and unclamping of the catheter begins to reestablish normal bladder function and capacity.

A nurses colleague has applied an incontinence pad to an older adult patient who has experienced occasional episodes of functional incontinence. What principle should guide the nurses management of urinary incontinence in older adults?

Urinary incontinence is not considered a normal consequence of aging.

A group of students are reviewing information about disorders of the bladder and urethra. The students demonstrate understanding of the material when they identify which of the following as a voiding dysfunction? a) Cystitis b) Bladder stones c) Urethral stricture d) Urinary retention

Urinary retention Urinary retention and urinary incontinence are voiding dysfunctions, temporary or permanent alterations in the ability to urinate normally. Cystitis is an infectious disorder. Bladder stones and urethral stricture are obstructive disorders.

Which of the following is a strategy to promote urinary continence? a) Implement a low fiber diet b) Take diuretics after 4 PM c) Use caffeine in moderation d) Void regularly, 5 to 8 times a day

Void regularly, 5 to 8 times a day Strategies to promote urinary continence include increasing awareness of the amount and timing of all fluid intake; avoid taking diuretics after 4 PM; avoiding bladder irritants, such as caffeine, alcohol, and aspartame (NutraSweet); taking steps to avoid constipation by drinking adequate fluids, eating a well-balanced diet high in fiber, exercising regularly, and taking stool softeners if recommended; and voiding regularly, 5 to 8 times a day (about every 2 to 3 hours).

If an indwelling catheter is necessary, the nursing interventions that should be implemented to prevent infection include a) placing the catheter bag on the patient's abdomen when moving the patient. b) using sterile technique to disconnect the catheter from tubing to obtain urine specimens. c) using clean technique during insertion. d) performing meticulous perineal care daily with soap and water.

performing meticulous perineal care daily with soap and water. Cleanliness of the area will reduce potential for infection. Strict aseptic technique must be used during insertion of a urinary bladder catheter. The nurse must maintain a closed system and use the catheter's port to obtain specimens. The catheter bag must never be placed on the patient's abdomen unless it is clamped because it may cause backflow of urine from the tubing into the bladder.

The nurse is assessing a patient admitted with renal stones. During the admission assessment, what parameters would be priorities for the nurse to address? Select all that apply

x) Dietary history x) Family history of renal stones x) Medication history

A patient has been admitted to the medical unit with a diagnosis of ureteral colic secondary to urolithiasis. When planning the patients admission assessment, the nurse should be aware of the signs and symptoms that are characteristic of this diagnosis? Select all that apply.

x) Hematuria x) Urinary frequency x) Acute pain

A 76-year-old with a diagnosis of penile cancer has been admitted to the medical floor. Because the incidence of penile cancer is so low, the staff educator has been asked to teach about penile cancer. What risk factors should the educator cite in this presentation? Select all that apply.

x) Phimosis x) Increasing age x) Lack of circumcision

A clinic nurse is providing preprocedure education for a man who will undergo a vasectomy. Which of the following measures will enhance healing and comfort? Select all that apply.

x) Wearing a scrotal support garment x) Using sitz baths Applying ice bags intermittently to the scrotum for several hours after surgery can reduce swelling and relieve discomfort, and is preferable to the application of heat. The nurse advises the patient to wear snug, cotton underwear or a scrotal support for added comfort and support. Sitz baths can also enhance comfort. Extended bed rest is unnecessary, and sexual activity can usually be resumed in 1 week.

The nurse is evaluating the effectiveness of discharge teaching for a client with an oxalate urinary stone. Which statement by the client indicates the need for further teaching by the nurse? Select all that apply. a) "I will never have another urinary stone again." b) "Tylenol is best to control my pain." c) "I'm so glad I don't have to make any changes in my diet." d) "I need to drink eight to ten glasses of water every day." e) "I need to take allopurinol."

• "I'm so glad I don't have to make any changes in my diet." • "Tylenol is best to control my pain." • "I will never have another urinary stone again." • "I need to take allopurinol." Nonsteroidal anti-inflammatory drugs are used to treat renal stone pain. Oxalate-containing foods should be avoided. Fluid intake should total 2 to 3 liters, if not contraindicated. Allopurinol (Zyloprim) is prescribed for uric acid stones. Recurrence of stones occurs in about half of individuals.

Which nursing diagnosis is appropriate for the client with a new ileal conduit? Select all that apply. a) Deficient knowledge: management of urinary diversion b) Urinary retention c) Risk for impaired skin integrity d) Disturbed body image e) Chronic pain

• Deficient knowledge: management of urinary diversion • Disturbed body image • Risk for impaired skin integrity Deficient knowledge, disturbed body image, and risk for impaired skin integrity are expected problems for the client with a new ileal conduit. Urinary retention and chronic pain are not expected client problems.


संबंधित स्टडी सेट्स

Chapter 5: Chain Reactions, Nuclear Reactors, and Atomic Bombs

View Set

Chapter 24 Module 2: Section 24.02

View Set

chpt 6: inventory & cost of goods sold

View Set

Standard 4- Learning Environments and Instructional Practices

View Set

Global Business BA310 Chapter 4 Exam

View Set

Chapter 1: Citizenship in Our Changing Democracy

View Set

What is a Computer Vocabulary Terms

View Set

Chapter 1 - The Anthropological Perspective On The Human Condition

View Set

Exam 3: Respiratory/digestive (part 3)

View Set

Chapter 30: Diabetes Mellitus 5-8

View Set

Chapter 9: Production and Operations Management

View Set

Organizational Behavior-Vanessa Bohns-Prelim 1

View Set

مصادر الثقافة الاسلامية ( القرآن الكريم)

View Set